ATI: Urinary & Bowel Elimination

Lakukan tugas rumah & ujian kamu dengan baik sekarang menggunakan Quizwiz!

You are caring for a patient whose primary care provide has written an order for "enemas until clear." Which of the following explanations to the patient about this procedure is correct?

"I will administer up to three enemas until there are no more pieces of stool in enema return." Explanation: A nurse would not be able to determine if the entire intestinal tract is clear.

When deciding whether to do invasive or noninvasive diagnotic bowel testing...

****NON INVASIVE also take presedence over INVASIVE** 1) fecal occult blood test 2) barium study 3) endoscopic exams

Stool Collection

***ASEPTIC TECHNIQUE*** - salmonella, e.coli, bacteria, c. diff (r/t antibiotics)

Physical Inspection of Abdomen

- Inspection: symmetrical, masses, distention, scars - Auscultation: bowel sounds, freq/character, clicks, flatus; are sounds hypoactive, hyperactive, absent or infrequent Percussion: resonant/tympany; dull = mass - Palpation: tenderness, masses, enlargements, resistance ***PULSING = abnormal; should be noted; aneurysm maybe

Patient Teaching with Stoma

- encourage to look at the stoma but respect not being to do so yet - avoid high fiber and smelly foods (dk green vegs) drink LOTS of water teach about meds resume daily activities and sexual relations

Methods for Emptying Colon

- enema - suppositories - oral lavage (GoLYTELY) - digital removal of stool

Large Intestine: Primary Organ for Elimination

- extends from ileocecal valve to anus FUNCTIONS: absorb water manufacture vitamins form feces expulsion of feces

Which of the following are included in the nursing plan of care to prevent adverse effects when caring for a patient with a nasogastric tube in place for gastric decompression?

-Irrigate with saline. -Measure the length of the exposed tube. -Measure the pH of the aspirated tube contents. -Administer frequent oral hygiene.

Nursing measures for Patients with diarrhea

-answer call light immediately -remove cause of diarrhea -obtain MD order for impaction removal -give special care to anal region

High risk for Constipation

-bedrest -reduced fluids -depression - CNS or local lesions that cause pain

colostomy care

-bright red -free of odor -empty bag when 1/3 full and replace 3-7 days -inspect site regularly -keep clean and dry -I&O -self-care education -psychosocial care and encouragement

wireless capsule endoscopy

-capsule with small camera is swallowed -pictures taken for 8 hours -2 pictures/second -expelled in 24-48 hours -no biopsies -can see any abnormalities non-invasively

Types of Enemas

-cleansing -retention -return flow

teaching pt about colostomies

-community resources -encourage pt to avoid high fiber foods -drink 2 quarts of water daily -teach about medications -teach about odor control (dark green vegetables) -resume normal activity

esophagogastroduodenoscopy

-consent -NPO 6-12 hr -sedated and local anesthetic -NPO until gag reflex returns -s/s of perforation

Colonoscopy

-consent -clear liquid 24-48hr before -2 day bowel prep (Dulcolax) -enema 1 day prior -sedation -flatulence -diet resumed when sedation wears off -bowel perforation

variables influencing bowel elimination

-developmental considerations -daily patterns -food and fluid -activity and muscle tone -lifestyle and psychological variables -pathologic conditions -medications -diagnostic studies -surgery and anesthesia

Variables affecting Bowel Elimination

-developmental considerations -daily patterns -food and fluid - activity and muscle tone - lifestyle, psychological variables - pathologic conditions - meds - diagnostic studies - surgery and anesthesia

sigmoidoscopy

-informed consent -light meal and 2 fleet enemas -no sedation -flatulence -s/s of perforation -biopsy could have some rectal bleeding

bowel training programs

-manipulate factors within pt control -food and fluid intake, exercise and defecation times -eliminate soft stool at regular intervals

Nurse interventions for Stoma Care

-soap and water ointments, suppositories, astringent = MD order

Hx of bowel elimination

-usual patterns -aids in elimination -recent changes in bowel elimination -problems with elimination -presence of artificial orifices

Patient guidelines for stool collection

-void first DEFECATE INTO CONTAINER; don't place toilet paper in there - call nurse when through

Mass peristatalic sweeps occur from ___ to ____ times every 24 hours in people

1 to 4

oil retention enema

150-200mL -lubricates stool and intestinal mucosa -30 min

paralytic ileus

24-48 hrs after anesthesia -NPO until peristalsis returns

soap suds enema

500-1000 mL -distend intestines, irritate mucosa and soften stool -10-15 min -rectal mucosa irritation or damage

hypertonic enema

70-130 mL -distend intestine, irritate intestinal mucosa -5-10 min -sodium retention -not for renal impairment

how much water is absorbed daily by the intestinal tract

800 to 1,000 mL

Which of the following patients is most likely to require interventions in order to maintain regular bowel patterns? a) A patient whose neuropathic pain requires multiple doses of opioids each day b) A patient who has a history of atrial fibrillation requiring daily anticoagulants c) A patient with hypertension who takes a diuretic and -adrenergic blocker each morning d) A 59-year-old woman who has recently begun hormone replacement therapy

A patient whose neuropathic pain requires multiple doses of opioids each day

During removal of a fecal impaction, which of the following could occur Because of vaginal stimulation? A) Bradycardia B)Atelectasid C) Tachycardia D) Cardiac tamponade

A) Bradycardia Removing a fecal impaction manually may result in stimulation of the vaginal nerve and resulting bradycardia

Which of the following is a true statement about the effects of medication on bowel illumination? A) Diarrhea commonly occurs with amoxicillin clavulanate use B) Anticoagulants cause a white discoloration of the stool C) Narcotic analgesics increased Gastrointestinal mobility D) Iron salts in pair digestion and cause a green store

A) Diarrhea commonly occurs with amoxicillin clavulanate use Anticoagulants may result in the store having a pink to red to black appearance, whereas iron salts also cause a black stool. Narcotic analgesics decrease gastric mobility.

Which of the following would be a common nursing diagnosis for patient with an ileostomy? A) Disturbed body image B) Constipation C) Delayed growth and development D) Excess fluid volume

A) Disturbed body image Constipation does not occur with the Ileostomy because the drainage is liquid. Growth and development are not affected by the formation of an Ileostomy. Excess fluid Byam is unlikely to occur because the drainage is liquid and probably continual

A barium Enema should be done before an upper gastrointestinal series because of which of the following? A) Retained barium may cloud the colon B)barium Can cause lower Gastrointestinal bleeding C) The physicians orders are in that sequence D)barium Is absorbed readily in the lower intestine

A) Retained barium may cloud the colon The barium And I'm should always perceive the upper gastrointestinal series because retained barium from the latter may take several days to pass through the gastrointestinal tract and may cloud anatomic detail on the barium enema studies

Which class of laxative acts by causing the stool to absorb water and swell? A)Bulk-forming B)Emollient C)Lubricant D)Stimulant

A)Bulk-forming Emollients Lubricate the stool; Lubricants soften the stool, making it easier to pass: and Stimulants promotes peristalsis by irritating the intestinal mucosa or stimulating nerve ending in the intestinal wall

A nurse is preparing to administer a cleaning enema to an adult client in preparation for a diagnostic procedure. Which of the following steps should the nurse take? (Select all that apply.) A. Warm the enema solution prior to instillation B. Position the client on the left side with the right leg flexed forward C. Lubricate the rectal tube or nozzle D. Slowly insert the rectal tube about 5cm (2in) E. Hand the enema container 61cm (24in) above.

A, B, C A. The nurse should warm the enema solution because cold fluid can cause abdominal cramping, and hot fluid can injure the intestinal mucosa B. The nurse should place the client in this position to promote a downward flow of solution by gravity along the natural anatomical curve of the sigmoid colon. C. The nurse should lubricate the tubing to prevent trauma or irritation to the rectal mucosa

Which of the following are included in the nursing plan of care to prevent adverse effects when caring for patients with a nasogastric tube in place for gastric decompression's? Select all that apply. A) Irrigation with Saline B) Measure the length of exposed tube C) Measure the pH of the aspirated tube contents D) Administer frequent oral hygiene

A,B,C,D After checking placement, NGT should be Irrigated with 30 to 60 mL of normal saline to maintain patency. The frequency is determined by facility policy, medical order, and nursing judgment. The length of the exposed tubes should be measured after insertion and routinely thereafter, as part of the assessment to verify placement and ensure the tube has not dislodged. Measuring the pH of aspirated two contents is one way to validate to placement in the intestinal tract. The other methods is to visually Assess aspirate to confirm gastric contents. Patients with NGTs Often experience discomfort related to irritation to nasal and throat mucosa And drying of the mucous membranes. Frequent oral hygiene should be administered as well as applying lubricant to the lips

Nursing Measures for Diarrhea

ALWAYS answer call bell immediately remove the cause of the diarrhea if possible (meds, etc) if impaction, get MD order for exam take care of the area around the anus

If a patient was instructed to avoid foods that may have a laxative effect, the nurse would advise the patient to avoid which of the following foods?

Alcohol.

A bowel training program includes which of the following?

Allowing ample time for evacuation.

A nurse is performing a physical assessment of a patient's abdomen. What is the normal sound that should be heard using the technique of percussion? A. Tympany B. Dullness C. Hyperresonance D. Gurgling

Answer: A. Tympany Rationale:Resonance or tympany is a normal sound expected when percussing the abdomen.

Affect of meds on stool

Aspirin, anticoags = pink to red to black - CALL MD Iron salts: black stool Antacids: white discoloration or speckling Antibiotics: green-gray color

Which of the following is an appropriate nursing action to promote regular bowel habits?

Assisting the patient to as normal position as possible to defecate.

If the patient was instructed to avoid foods that may have a laxative effect, the nurse would advise the patient to avoid which of the following foods? A) Chinese B) Alcohol C) Eggs D) Pasta

B) Alcohol All the foods listed as such alcohol have a constipating effect

Your patient complains of an excessive flatulence. When reviewing your patients dietary intake, which foods, if eaten regularly, would you identify as possibly responsible? A) Meet B) Cauliflower C) Potatoes D) Ice cream

B) Cauliflower Cauliflower is a gas producing food that relates in flatulence

Nurses should recommend avoiding the habitual use of laxatives. Which of the following is the rationale for this? A) They will cause a fecal impaction B) They will cause chronic constipation C) They change the pH of the Gastrointestinal track D) They inhibit the intestinal enzymes

B) They will cause chronic constipation Habitual use of laxatives is the most common cause of chronic constipation

Mr. T is nervous about a colonoscopy scheduled for tomorrow. The nurse describes the test by explaining that it allows which of the following? A) Visual examination of the esophagus and stomach B) Visual examination of the large intestine C) Radiographic examination of the large intestine D) Fluoroscopic examination of the small intestine

B) Visual examination of the large intestine An esophagogastroduodenoscopy Allows visual examination of the esophagus and stomach. The radiographic examination of the large intestine refers to a barium enema, and a fluoroscopic Examination of the small intestines refers to an upper gastrointestinal series

A nurse is caring for a client who has had diarrhea for 4 days. When assessing the client, the nurse should expect which of the following findings (select all that apply.) A. Bradycardia B. Hypotension C. Elevated temperature D. Poor skin turgor E. Peripheral edema

B, C, D Prolonged diarrhea leads to dehydration, the nurse should expect client to have: -decreased BP -increased temp. -poor skin turgor

Which of the following direct visualization tests uses a long, flexible, fiber-optic-lighted scope to visualize the rectum, colon, and distal small bowel? A. Esophagogastroduodenoscopy B. Colonoscopy C. Sigmoidoscopy D. UGI series

B. Colonoscopy

A nurse is talking with a client who reports constipation. When the nurse discusses dietary changes that can help prevent constipation, which of the following foods should the nurse recommend? A. Mac & cheese B. Fresh fruit and whole wheat toast C. Bread pudding and yogurt D. Roast chicken and white rice

B. Fresh fruit and whole wheat toast A high-fiber diet promotes normal bowel elimination. The nurse should recommend the client consume fresh fruits vegetables with whole-grain carbs to provide the highest fiber option.

When the nurse performs a Hemoccult test on a stool specimen, blood in the stool will change the color on the test paper to

Blue

When the nurse performs a Hemoccult test on a stool specimen, blood in the stool will change the color on the test paper to a) Brown b) Red c) Green d) Blue

Blue

An elderly woman who is incontinent of stool following a cerebrovascular accident will have the following nursing diagnosis a) Diarrhea related to tube feedings as evidenced by hyperactive bowel sounds and urgency b) Fecal retention related to loss of sphincter control and diminished spinal cord innervation related to hemiparesis c) Constipation related to physiologic condition involving the deficit in neurologic innervation as evidenced by fecal incontinence d) Bowel incontinence related to loss of sphincter control as evidenced by inability to delay the urge to defecate

Bowel incontinence related to loss of sphincter control as evidenced by inability to delay the urge to defecate

During removal of a fecal impaction, which of the following could occur because of vagal stimulation?

Bradycardia.

Which class of laxative acts by causing the stool to absorb water and swell?

Bulk-forming.

Mr. Jay has a fecal impaction. The nurse correctly administers an oil-retention Enema by doing which of the following? A) Administering a large volume solution 500 to 1000 ml B) Mixing milk and molasses and equal parts for an enema C) Instructing the patient to retain the enema for at least 30 seconds D) Administering the enema while the patient is sitting on a toilet

C) Instructing the patient to retain the enema for at least 30 seconds The usual amount of solution administered with a retention Enema is 150 to 200 mL for an adult. The milk and molasses mixture is a carminative enema That helps to expel flats, As does the Harrison flush procedure

When explaining the action of a hyper tonic solution enema, the nurse incorporates which of the following as the basis for action? A) bowel mucosa irritations B) Diffusion of water out of colon C) Osmosis of water into colon D) Softening of fecal contents

C) Osmosis of water into colon Hypertonic solutions draw water into the colon a By osmosis that's stimulating the defecation reflex. Orrills solutions soften fecal contents, and soup solutions distend the intestines and irritate the bowel mucosa

Your patient complains of excessive flatulence. When reviewing the patient's dietary intake, which food, if eaten regularly, would you identify as possibly responsible?

Cauliflower

Stool and Developmental Considerations

Child: depend on formula or breat feedings Toddler: physiologic maturity is first priority for bowel training Child/adolescent/adult: defecation patterns vary in quantity, frequency, and rhythmicity Older Adult: constipation is chronic problem; diarrhea andn fecal incontinence may result from physiologic or lifestyle changes

You are preparing to apply a fecal incontinence pouch. Arrange the following steps in the correct order.

Cleanse entire perianal area and pat dry. Apply skin protectant and allow it to dry. Separate buttocks and apply the pouch to the anal area. Attach the pouch to a urinary drainage bag. Hang the drainage bag below the patient.

A patient scheduled for a colonoscopy is scheduled to receive a hypertonic enema prior to the procedure. A hypertonic enema is classified as which type of enema? a) Cleansing enema b) Retention enema c) Carminative enema d) Return-flow enema

Cleansing

Foods affecting bowel elimination

Constipating Foods: cheese, lean meat, eggs, pasta Foods with laxative effect: fruits/veg, bran, chocolate, alcohol, coffee Gas producing: onions, cabbage, beans, cauliflower

The nursing instructor informs a student nurse that a patient she is caring for has a chronic neurologic condition and the condition decreases the patient's peristalsis. What nursing diagnosis is the most likely risk for this patient? a) Diarrhea b) Excessive fluid volume c) Deficient fluid volume d) Constipation

Constipation

A woman age 76 years has informed the nurse that she has begun using over-the-counter laxatives because her friend told her it was imperative to have at least one bowel movement daily. How should the nurse best respond to this client's statement?

Correct response: "Actually, people's bowel patterns can vary a lot and some people don't tend to go every day." Explanation: Elimination patterns vary widely among individuals, and the expectation of a daily bowel movement is not realistic for many healthy people. This client may not require pharmacologic interventions. Reference: Taylor, C., Lynn, P., & Bartlett, J., Fundamentals of Nursing, 9th ed., Philadelphia, Wolters Kluwer, 2019, Chapter 38: Bowel Elimination, p. 1420.

A nurse has auscultated the abdomen in all four quadrants for 5 minutes and has not heard any bowel sounds. How would this be documented?

Correct response: "All four abdominal quadrants auscultated. Inaudible bowel sounds." Explanation: In the correct response, the nurse has documented what was done during the assessment and has noted that bowel sounds are inaudible. "Auscultated abdomen for bowel sounds. Bowel not functioning" is not appropriate as the nurse has diagnosed that the bowel is not functioning which is a medical diagnosis. The documentation lacks the assessment. "Bowel sounds auscultated. Client has no bowel sounds" is not appropriate does not indicate where bowel sounds were auscultated. "Client may have bowel sounds, but they can't be heard" is a subjective statement and does not document the assessment. Reference: Taylor, C., Lynn, P., & Bartlett, J., Fundamentals of Nursing, 9th ed., Philadelphia, Wolters Kluwer, 2019, Chapter 38: Bowel Elimination, p. 1452.

A client with no significant medical history reports experiencing diarrhea over the past week. Which assessment question will the nurse ask? Select all that apply.

Correct response: "Have you started a new medication?" "What are your normal bowel habits?" "Do you use laxatives?" Explanation: The nurse will ask about new medications because these can often cause diarrhea; what the client's normal bowel habits are like, to establish a baseline; and whether the client is using laxatives, which can contribute to diarrhea. Rectal fullness and stool that is difficult to pass are associated with constipation. Reference: Taylor, C., Lynn, P., & Bartlett, J., Fundamentals of Nursing, 9th ed., Philadelphia, Wolters Kluwer, 2019, Chapter 38: Bowel Elimination, pp. 1421-1425.

A nurse is caring for a client whose primary care provider has written an order for "enemas until clear." Which explanation to the client about this procedure is correct?

Correct response: "I will administer enemas until the enema return is without stool." Explanation: "Enemas until clear" means that the nurse would administer enemas until no more stool is noted on output. A nurse would not be able to determine if the entire intestinal tract is clear. Administering three enemas is not what the prescriber ordered. Consuming clear liquids does not impact the use of enemas. The enema may not be part of the client's discharge instructions. Reference: Taylor, C., Lynn, P., & Bartlett, J., Fundamentals of Nursing, 9th ed., Philadelphia, Wolters Kluwer, 2019, Chapter 38: Bowel Elimination, pp. 1438-1439.

An older adult client tells the nurse, "I give myself a mineral oil enema every day." What is the appropriate nursing response?

Correct response: "Mineral oil enemas can interfere with absorption of fat-soluble vitamins." Explanation: The nurse will caution the client that self-administration of mineral oil to relieve constipation can interfere with absorption of fat-soluble vitamins. The nurse can then further discuss the reason the client is performing this treatment and determine other appropriate interventions to relieve constipation. Reference: Taylor, C., Lynn, P., & Bartlett, J., Fundamentals of Nursing, 9th ed., Philadelphia, Wolters Kluwer, 2019, Chapter 38: Bowel Elimination, p. 1435.

The nurse needs to collect stool for occult blood testing from an 8-month-old client. The parent asks if the specimen for testing can be collected from the child's diaper. What is the best response by the nurse?

Correct response: "Only if the stool has not been contaminated by urine." Explanation: Stool can be collected from a diaper for occult blood testing only if the stool has not been contaminated by urine. It does not matter whether the diaper is disposable or cloth. The type of developer does not make a difference as all are used to test for occult blood. Reference: Taylor, C., Lynn, P., & Bartlett, J., Fundamentals of Nursing, 9th ed., Philadelphia, Wolters Kluwer, 2019, Chapter 38: Bowel Elimination, pp. 1428-1429.

The nurse has presented an educational in-service about caring for clients who have newly created ostomies. The nurse asks participants, "How will you know when a client begins to accept the altered body image?" Which responses by participants indicates a correct understanding of the material? Select all that apply

Correct response: "The client is willing to look at the stoma." "The client makes neutral or positive statements about the ostomy." "The client expresses interest in learning self-care." Explanation: Reference: Taylor, C., Lynn, P., & Bartlett, J., Fundamentals of Nursing, 9th ed., Philadelphia, Wolters Kluwer, 2019, Chapter 38: Bowel Elimination, p. 1447.

The nurse is caring for a client who reports constipation and is presently in the bathroom attempting to have a bowel movement. The client presses the call bell and tells the nurse that about feeling dizzy. What education should the nurse provide the client about this condition?

Correct response: "This occurs when bearing down and decreasing blood flow to the heart; when you stop, the blood flow will return in a larger amount." Explanation: Reference: Taylor, C., Lynn, P., & Bartlett, J., Fundamentals of Nursing, 9th ed., Philadelphia, Wolters Kluwer, 2019, Chapter 38: Bowel Elimination, p. 1420.

A client has been given fecal occult blood test (FOBT) testing supplies. What teaching will the nurse provide about the purpose for this test?

Correct response: "This test detects heme, an iron compound in blood within the stool." Explanation: The nurse will teach that the FOBT detects heme. It does not test for allergic foods, nor does it test for infection. The fecal immunochemical test (FIT) test results have a high rate of specificity for colorectal cancer. Reference: Taylor, C., Lynn, P., & Bartlett, J., Fundamentals of Nursing, 9th ed., Philadelphia, Wolters Kluwer, 2019, Chapter 38: Bowel Elimination, pp. 1428-1429.

The nurse has provided instructions to a client having a fecal immunochemical test (FIT). The client states, "I am menstruating right now. Is it okay to still do the test?" What is the best response by the nurse?

Correct response: "Wait to do the test 3 days after your finish menstruating." Explanation: The client should be sure to postpone the test until 3 days after cessation of menstruation. If not, the client may experience a false-positive test. Reference: Taylor, C., Lynn, P., & Bartlett, J., Fundamentals of Nursing, 9th ed., Philadelphia, Wolters Kluwer, 2019, Chapter 38: Bowel Elimination, p. 1429.

The nurse is preparing to administer a large-volume enema to an adult client. How far should the nurse insert the tubing into the rectum?

Correct response: 3 in (7.5 cm) Explanation: The tube should be inserted past the external and internal sphincters, approximately 3 in (7.5 cm). Further insertion, such as 5 in (12.5 cm), may damage intestinal mucous membrane. If the tube is inserted less than 3 in (7.5 cm), then the enema solution will not make it into the rectum but will seep out during the administration of the enema. Reference: Taylor, C., Lynn, P., & Bartlett, J., Fundamentals of Nursing, 9th ed., Philadelphia, Wolters Kluwer, 2019, Chapter 38: Bowel Elimination, p. 1419.

The nurse is providing health teaching for four clients. Which client should consider a colonoscopy screening?

Correct response: 50-year-old client with a family history of polyps Explanation: The nurse will teach that the 50-year-old client with a family history of polyps should consider a colonoscopy screening. Screenings should start at 50 years old and continue every 10 years thereafter. Other answers are incorrect. Reference: Taylor, C., Lynn, P., & Bartlett, J., Fundamentals of Nursing, 9th ed., Philadelphia, Wolters Kluwer, 2019, Chapter 38: Bowel Elimination, pp. 1428-1429.

The nurse is replacing a client's ileostomy appliance and has identified that the diameter of the stoma is 3.5 cm. The nurse has trimmed the flange of the new appliance to a diameter of 7 cm. What will be the most likely outcome of the nurse's action?

Correct response: A risk that the peristomal skin will become excoriated Explanation: An appliance that is too large will expose peristomal skin to digestive enzymes, leading to skin breakdown. It will not fit as securely as it should, but this does not necessarily mean that it will need to be changed daily. Prolapse is not related to the way that the appliance is sized or trimmed. Reference: Taylor, C., Lynn, P., & Bartlett, J., Fundamentals of Nursing, 9th ed., Philadelphia, Wolters Kluwer, 2019, Chapter 38: Bowel Elimination, pp. 1468-1473.

A student nurse is preparing to administer a client's ordered large-volume enema. What action should the nurse perform during this skill?

Correct response: Administer the solution gradually over 5 to 10 minutes. Explanation: Large-volume enemas should be given over a 5- to 10-minute time frame. The solution should be warm, but warming for a specific time period in a microwave could result in overheating. It is not always necessary or possible for the client to attempt a bowel movement prior to the procedure. If performed correctly, the procedure should not necessitate analgesia. Reference: Taylor, C., Lynn, P., & Bartlett, J., Fundamentals of Nursing, 9th ed., Philadelphia, Wolters Kluwer, 2019, Chapter 38: Bowel Elimination, pp. 1453-1456.

An older adult client is in the hospital following an intestinal diversion with an ileostomy on the right upper quadrant and a mucous fistula. What is the most important nursing action in the care of this client?

Correct response: Assess the color of the stoma. Explanation: A bluish or dark stoma indicates impaired circulation to the stoma. This requires intervention to improve circulation to avoid permanent damage to the stoma. Applying the device would be secondary after the assessment. A stoma irrigation is not a priority in the care of the client. Having the client perform self stoma care would be one of the last interventions provided prior to discharge from the hospital after assessing for readiness. Reference: Taylor, C., Lynn, P., & Bartlett, J., Fundamentals of Nursing, 9th ed., Philadelphia, Wolters Kluwer, 2019, Chapter 38: Bowel Elimination, p. 1446.

A nurse is caring for a client with an NG tube attached to continuous suction. The nurse observes that the tube is connected to the wall suction, but it is not draining. What is the nurse's best action?

Correct response: Attempt to irrigate the NG tube with water or normal saline. Explanation: An NG tube that is not draining should normally be irrigated. Turning the suction off and on is less likely to be effective, and it may be unsafe to leave the suction turned off for half an hour. Digestive enzymes are not used on NG tubes that are used for suction. Removing the NG tube would be an action of last resort. Reference: Taylor, C., Lynn, P., & Bartlett, J., Fundamentals of Nursing, 9th ed., Philadelphia, Wolters Kluwer, 2019, Chapter 38: Bowel Elimination, pp. 1465-1467.

A client is preparing for a fecal occult blood test. What teaching will the nurse provide regarding vitamin C three days before testing?

Correct response: Avoid more than 250 mg Explanation: The nurse will teach the client to avoid taking more than 250 mg of vitamin C two to three days before testing, and not to consume citrus fruits or juices. Therefore, the other answers are incorrect. Reference: Taylor, C., Lynn, P., & Bartlett, J., Fundamentals of Nursing, 9th ed., Philadelphia, Wolters Kluwer, 2019, Chapter 38: Bowel Elimination, p. 1429.

Removal of a client's NG tube has been ordered. Which action should the nurse perform during this intervention?

Correct response: Before removing the tube, discontinue suction and separate the tube from suction. Explanation: When removing the tube, the nurse should discontinue the suction and separate the tube from suction to allow for its unrestricted removal. The client should be placed in a 30- to 45-degree position. The tube should be flushed with 10 mL of water or normal saline solution and should be removed as the client holds his or her breath. Reference: Taylor, C., Lynn, P., & Bartlett, J., Fundamentals of Nursing, 9th ed., Philadelphia, Wolters Kluwer, 2019, Chapter 38: Bowel Elimination, pp. 1463-1465.

A nurse prepares to insert a nasointestinal tube to provide nutrition to a client. Which guideline is recommended for this procedure?

Correct response: Begin by measuring from the tip of the client's nose to the earlobe to the xiphoid process. Explanation: To insert a nasointestinal tube, the nurse should measure the tube from the tip of nose to the earlobe and from the earlobe to the xiphoid process and add 8 to 10 in (20 to 25 cm) for intestinal placement. The client should be placed on his or her right side. Analgesia is not normally required in anticipation of placement. Reference: Taylor, C., Lynn, P., & Bartlett, J., Fundamentals of Nursing, 9th ed., Philadelphia, Wolters Kluwer, 2019, Chapter 38: Bowel Elimination, pp. 1457-1462.

An older adult woman who is incontinent of stool following a cerebrovascular accident will have which nursing diagnosis?

Correct response: Bowel Incontinence related to loss of sphincter control, as evidenced by inability to delay the urge to defecate Explanation: The most appropriate nursing diagnosis addresses the client's fecal incontinence, related to loss of sphincter control innervation. Reference: Taylor, C., Lynn, P., & Bartlett, J., Fundamentals of Nursing, 9th ed., Philadelphia, Wolters Kluwer, 2019, Chapter 38: Bowel Elimination, p. 1443.

A nurse is irrigating a client's nasogastric tube. Place the following steps in the correct order. Use all options.

Correct response: Check placement of the nasogastric tube. Draw up 30 mL of irrigation solution into a syringe. Clamp the nasogastric tube near the connection site. Hold the syringe upright, and gently insert the irrigant. Hold the end of the nasogastric tube over an emesis basin. Inject air into the blue air vent. Explanation: Checking placement before the instillation of fluid is necessary to prevent accidental instillation into the respiratory tract if the tube has become dislodged. Drawing up the specified amount of solution into a syringe ensures delivery of the proper amount of irrigant through the tube. Clamping prevents leakage of gastric fluid. Gentle insertion of saline solution (or gravity insertion) is less traumatic to gastric mucosa. Return flow may be collected in an irrigating tray or other available container and measured. This amount needs to be subtracted from the irrigant to record the true nasogastric drainage. Following irrigation, the blue air vent is injected with air to keep it clear. Reference: Taylor, C., Lynn, P., & Bartlett, J., Fundamentals of Nursing, 9th ed., Philadelphia, Wolters Kluwer, 2019, Chapter 38: Bowel Elimination, pp. 1465-1468.

The nurse is administering a cleansing enema when the client reports cramping. What is the appropriate nursing action?

Correct response: Clamp the tube for a brief period and resume at a slower rate. Explanation: Cramping can occur with the administration of a cleansing enema. The nurse will clamp the tube for a brief period while the client takes deep breaths, and then resume the infusion, possibly at a slower rate. Other choices are incorrect, as these do not facilitate administration of the enema while providing comfort measures. Reference: Taylor, C., Lynn, P., & Bartlett, J., Fundamentals of Nursing, 9th ed., Philadelphia, Wolters Kluwer, 2019, Chapter 38: Bowel Elimination, p. 1455.

A nurse is collecting a stool specimen of a client suspected of having Clostridium difficile. Which guideline is recommended for this procedure?

Correct response: Collect 15 to 30 mL of the client's liquid stool. Explanation: Usually, 1 in (2.5 cm) of formed stool or 15 to 30 mL of liquid stool is sufficient; this client is more likely to have liquid stool. If portions of the stool include visible blood, mucus, or pus, include these with the specimen. Also be sure that the specimen is free of any barium or enema solution. Because a fresh specimen produces the most accurate results, send the specimen to the laboratory immediately. Reference: Taylor, C., Lynn, P., & Bartlett, J., Fundamentals of Nursing, 9th ed., Philadelphia, Wolters Kluwer, 2019, Chapter 38: Bowel Elimination, pp. 1428-1429.

A nurse is testing a client's stool specimen for occult blood. Which are responsibilities of the nurse for this testing? Select all that apply.

Correct response: Collecting the specimen Handling the specimen Transporting the specimen Teaching the client about the test Explanation: The nurse should follow facility protocol to collect, handle, and transport a specimen. It is very important to adhere to protocols and standards, collect the appropriate amount, use appropriate containers and media, and store and transfer the specimen within specified timelines. Client teaching is also an important part of specimen collection. The primary health care provider orders the test and plans medical treatment based on the results. Reference: Taylor, C., Lynn, P., & Bartlett, J., Fundamentals of Nursing, 9th ed., Philadelphia, Wolters Kluwer, 2019, Chapter 38: Bowel Elimination, pp. 1428-1429.

Which symptom is a known side effect of antibiotics?

Correct response: Diarrhea Explanation: A side effect of taking antibiotics is diarrhea. Constipation, fecal impaction, and abdominal bloating are not common side effects of antibiotics. Reference: Taylor, C., Lynn, P., & Bartlett, J., Fundamentals of Nursing, 9th ed., Philadelphia, Wolters Kluwer, 2019, Chapter 38: Bowel Elimination, p. 1437.

Which statement best explains why digital removal of stool is considered a last resort after other methods of bowel evacuation have been unsuccessful?

Correct response: Digital removal of stool may cause parasympathetic stimulation. Explanation: The procedure may stimulate a vagal response, which increases parasympathetic stimulation. The nurse does use digital removal as a last resort. It is an uncomfortable but necessary procedure for the client. Because clients are uncomfortable with fecal impaction, the client will consent for the procedure. Digital removal does not cause rebound diarrhea nor electrolyte loss. Reference: Taylor, C., Lynn, P., & Bartlett, J., Fundamentals of Nursing, 9th ed., Philadelphia, Wolters Kluwer, 2019, Chapter 38: Bowel Elimination, p. 1454.

The nurse is preparing to auscultate the bowel sounds of a client with a nasogastric tube in place set to low intermittent suction. How shall the nurse approach the assessment of bowel sounds and manage the nasogastric tube?

Correct response: Disconnect the nasogastric tube from suction during the assessment of bowel sounds. Explanation: If the client has a nasogastric tube in place, disconnect it from the suction during this assessment to allow for accurate interpretation of sounds. Allowing the low intermittent to continue during the assessment will interfere with the auscultation of the sounds. Disconnect of the tube can occur immediately and not for 1 hour prior to the assessment. Reference: Taylor, C., Lynn, P., & Bartlett, J., Fundamentals of Nursing, 9th ed., Philadelphia, Wolters Kluwer, 2019, Chapter 38: Bowel Elimination, p. 1426.

The nurse is administering magnesium citrate to a client with constipation. What mechanism of action would the nurse expect from this drug?

Correct response: Drawing water into the intestines to stimulate peristalsis Explanation: Magnesium citrate increases intestinal bulk by drawing water into the intestine and stimulating peristalsis. Chemical stimulation of peristalsis is promoted by stimulants such as bisacodyl. The stimulant promotes peristalsis by irritating the intestinal mucosa or stimulating nerve endings in the intestinal wall. Emollients such as mineral oil lubricate the intestinal tract and retard colonic absorption of water, softening the stool, and making it easier to pass. Bulk forming agents, such as psyllium husk, increase intestinal bulk to enhance mechanical stimulation of the intestine. Reference: Taylor, C., Lynn, P., & Bartlett, J., Fundamentals of Nursing, 9th ed., Philadelphia, Wolters Kluwer, 2019, Chapter 38: Bowel Elimination, p. 1435.

The nurse is caring for a client who is scheduled for an esophagogastroduodenoscopy (EGD). What action would the nurse take to prepare the client for this procedure?

Correct response: Ensure that the client fasts 6 to 12 hours before the test as per policy. Explanation: The nurse would ensure that the client fasted 6 to 12 hours before the test as per policy. The nurse would not provide a light meal before the test, nor administer two Fleet enemas for an EGD. The client would not ingest a gallon of bowel cleanser. The nurse would not give the client a barium contrast mixture to drink. Reference: Taylor, C., Lynn, P., & Bartlett, J., Fundamentals of Nursing, 9th ed., Philadelphia, Wolters Kluwer, 2019, Chapter 38: Bowel Elimination, p. 1429.

A nurse is caring for a client recovering from abdominal surgery who is experiencing paralytic ileus. The client has a nasogastric tube connected to suction. How often should the nurse irrigate this tube?

Correct response: Every 4 to 8 hours Explanation: The tube must be kept free from obstruction or clogging and is usually irrigated every 4 to 8 hours. Reference: Taylor, C., Lynn, P., & Bartlett, J., Fundamentals of Nursing, 9th ed., Philadelphia, Wolters Kluwer, 2019, Chapter 38: Bowel Elimination, pp. 1465-1468.

Which statement about ostomy irrigation is true?

Correct response: For some clients, regularly scheduled colostomy irrigation can be used to establish a predictable pattern of elimination. Explanation: For some clients, regularly scheduled colostomy irrigation can be used to establish a predictable pattern of elimination. This approach allows for the use of a small covering over the colostomy between irrigations, instead of a regular appliance. Daily irrigation is necessary to assure passage of stool from an ileostomy is not warranted as ileostomy do not require daily irrigation. A contract is not necessary to sign to use the equipment. Ostomy prolapse can be delayed by resting until the prolapse recedes and twice daily irrigation is not necessary. Reference: Taylor, C., Lynn, P., & Bartlett, J., Fundamentals of Nursing, 9th ed., Philadelphia, Wolters Kluwer, 2019, Chapter 38: Bowel Elimination, p. 1448.

A nurse is assessing a client who has recently had bowel surgery and will be receiving a nasogastric tube. Which finding would most likely contraindicate placement of a nasogastric (NG) tube by the nurse in this client?

Correct response: History of facial fractures Explanation: Clients with facial fractures or facial surgeries present a higher risk for misplacement of the tube into the brain. Many institutions require a health care provider to place NG tubes in these clients, which would contraindicate the nurse placing the tube. The nurse should assess the patency of the client's nares by asking the client to occlude one nostril and breathe normally through the other. However, the nurse does this to select the nostril through which air passes more easily, not because one nare being less patent than the other is a contraindication for NG tube placement by the nurse. Abdominal distention does not contraindicate NG tube placement. Monitoring bleeding in the gastrointestinal (GI) tract is one of the indications for NG tube placement, so bleeding in the GI tract is not a contraindication. Reference: Taylor, C., Lynn, P., & Bartlett, J., Fundamentals of Nursing, 9th ed., Philadelphia, Wolters Kluwer, 2019, Chapter 38: Bowel Elimination, pp. 1457-1462.

A nurse is teaching a student nurse how to manage unexpected events during the removal of a nasogastric tube. Which action should the nurse recommend?

Correct response: If within 2 hours after NG tube removal, the client's abdomen is showing signs of distention, notify the health care provider. Explanation: If within 2 hours after NG tube removal, the client's abdomen is showing signs of distention, notify the health care provider. The health care provider may order the nurse to replace the NG tube. If epistaxis occurs with removal of the NG tube, occlude both nares until bleeding has subsided and ensure the client is in an upright position. Petroleum jelly is not used to address pain during removal. The nurse cannot independently reinsert the NG tube. Reference: Taylor, C., Lynn, P., & Bartlett, J., Fundamentals of Nursing, 9th ed., Philadelphia, Wolters Kluwer, 2019, Chapter 38: Bowel Elimination, pp. 1463-1465.

The student nurse is preparing a presentation on how to perform a physical assessment on the abdomen. Place the assessment steps in the correct order.

Correct response: Inspection Auscultation Percussion Palpation Explanation: When assessing a client's abdomen, the correct order for assessment is inspection, auscultation, percussion, and palpation. Reference: Taylor, C., Lynn, P., & Bartlett, J., Fundamentals of Nursing, 9th ed., Philadelphia, Wolters Kluwer, 2019, Chapter 38: Bowel Elimination, p. 1426.

Which medication causes constipation?

Correct response: Iron supplements Explanation: A common side effect of iron supplements is constipation. Bisacodyl is a stool softener. Aspirin is an analgesic that does not typically cause constipation. Magnesium antacids help to decrease heartburn and do not typically cause constipation. Reference: Taylor, C., Lynn, P., & Bartlett, J., Fundamentals of Nursing, 9th ed., Philadelphia, Wolters Kluwer, 2019, Chapter 38: Bowel Elimination, p. 1422.

The nurse is assisting an older adult client into position for a sigmoidoscopy. Which position would the nurse place the client in?

Correct response: Left lateral Explanation: The left lateral or knee to chest position is the most common position for sigmoidoscopies or colonoscopies. If the client is not able to tolerate this position, Sims' position may also be used. The right lateral, prone or semi-Fowler's positions are not routinely used for this procedure. Reference: Taylor, C., Lynn, P., & Bartlett, J., Fundamentals of Nursing, 9th ed., Philadelphia, Wolters Kluwer, 2019, Chapter 38: Bowel Elimination, p. 1430.

The nurse is administering a large-volume cleansing enema to a client who reports severe cramping upon introduction of the enema solution. What would be the nurse's next action?

Correct response: Lower solution container and check temperature and flow rate. Explanation: The nurse's next action would be to lower the solution container and check the temperature and flow rate. Lowering the solution container decreases the pressure of the flow of the solution. The cramping could be related to the pressure of the flow, the temperature of the solution, or a high flow rate of the solution. The nurse would not place the client in a supine position, but in a low-Fowler's position or higher. The nurse would not remove the tube and check for any fecal contents. The nurse would not modify the amount and length of the administration, as this is not causing the severe cramping. Reference: Taylor, C., Lynn, P., & Bartlett, J., Fundamentals of Nursing, 9th ed., Philadelphia, Wolters Kluwer, 2019, Chapter 38: Bowel Elimination, p. 1453.

A nurse needs to administer an enema to a client to lubricate the stool and intestinal mucosa to make stool passage more comfortable. Which type of enema should the nurse administer?

Correct response: Oil-retention Explanation: Oil-retention enemas help to lubricate the stool and intestinal mucosa, making defecation easier. A hypertonic enema draws water into the colon, which stimulates the defecation reflex. Carminative enemas help to expel flatus from the rectum and relieve distention secondary to flatus. Anthelmintic enemas are administered to destroy intestinal parasites. Reference: Taylor, C., Lynn, P., & Bartlett, J., Fundamentals of Nursing, 9th ed., Philadelphia, Wolters Kluwer, 2019, Chapter 38: Bowel Elimination, Table 38-4, p. 1439.

A nurse is performing an abdominal assessment of a client before administering a large-volume cleansing enema. Which assessment technique would be performed last?

Correct response: Palpation Explanation: The abdominal assessment should be performed in the following sequence: inspection, auscultation, percussion, palpation. Reference: Taylor, C., Lynn, P., & Bartlett, J., Fundamentals of Nursing, 9th ed., Philadelphia, Wolters Kluwer, 2019, Chapter 38: Bowel Elimination, p. 1426.

Which medical diagnosis is most likely to necessitate testing for fecal occult blood?

Correct response: Peptic Ulcer Explanation: Any health problem that involves bleeding of the GI tract, such as peptic ulcer disease, may require fecal occult blood testing (FOBT). Constipation does not indicate a need for FOBT unless hardened stool is suspected of causing GI trauma. Similarly, GERD may require FOBT only if esophageal bleeding is suspected. Liver disease is not a common indication for FOBT. Reference: Taylor, C., Lynn, P., & Bartlett, J., Fundamentals of Nursing, 9th ed., Philadelphia, Wolters Kluwer, 2019, Chapter 38: Bowel Elimination, p. 1428.

A nurse attempts to administer a tap water enema to a client who is dehydrated and finds that the client cannot retain the enema for the prescribed amount of time. What nursing action would be appropriate for this client?

Correct response: Place the client on a bedpan in supine position while receiving the enema and elevate the head of the bed 30 degrees. Explanation: If the client cannot retain the enema solution for an adequate amount of time, place the client on the bedpan in a supine position while receiving the enema. Elevate the head of the bed 30 degrees for the client's comfort. If still unable to retain the solution, notify the physician. The nurse does not need to reposition the rectal tube but needs to assist the client by repeating the procedure with a slight variation. Reference: Taylor, C., Lynn, P., & Bartlett, J., Fundamentals of Nursing, 9th ed., Philadelphia, Wolters Kluwer, 2019, Chapter 38: Bowel Elimination, pp. 1439-1440.

In preparing a client to utilize fecal occult blood testing (FOBT) supplies, what teaching will the nurse provide?

Correct response: Refrain from eating red meat 3 days before testing. Explanation: The nurse will teach that the client should avoid eating red meat 3 days before testing, refrain from consuming citrus fruits or juices for 3 days before beginning the test, and to avoid certain raw vegetables 2-3 days prior to testing. Acetaminophen use is acceptable; nonsteroidal anti-inflammatory drugs (NSAIDs) must be avoided 7 days before self-collecting stool. Reference: Taylor, C., Lynn, P., & Bartlett, J., Fundamentals of Nursing, 9th ed., Philadelphia, Wolters Kluwer, 2019, Chapter 38: Bowel Elimination, pp. 1428-1429.

The nurse needs to collect a stool specimen for culture from a client. The client passed stool into the toilet instead of using the collection container. What is the next step for the nurse?

Correct response: Reinstruct the client on use of collection container for next bowel movement. Explanation: Stool should not be collected from the toilet due to contamination. The nurse should reinstruct the client on the use of a collection container for the next bowel movement. There is nothing to indicate that the test should be cancelled nor that the client needs a laxative administered. Reference: Taylor, C., Lynn, P., & Bartlett, J., Fundamentals of Nursing, 9th ed., Philadelphia, Wolters Kluwer, 2019, Chapter 38: Bowel Elimination, p. 1428.

The nurse is inserting a rectal tube to administer a large-volume enema. Which nursing action is performed correctly in this procedure?

Correct response: Slowly and gently insert the enema tube 3 to 4 in (7.5 to 10 cm) for an adult. Explanation: The nurse would slowly and gently insert the enema tube 3 to 4 in (7.5 to 10 cm) for an adult. The nurse would not position the client in a supine position, rather on the left side in the Sims' position. This position aids in the client's ability to retain the solution. The nurse would not introduce the solution quickly, as this will result in the client cramping. The nurse would administer the solution over 5 to 10 minutes, depending on the volume. The nurse would not encourage the client to hold the solution for at least 20 minutes, rather 5 to 15 minutes when the urge to defecate becomes strong. Reference: Taylor, C., Lynn, P., & Bartlett, J., Fundamentals of Nursing, 9th ed., Philadelphia, Wolters Kluwer, 2019, Chapter 38: Bowel Elimination, p. 1439.

The student nurse is administering a large-volume enema to a client. The client reports abdominal cramping. What should the student nurse do first?

Correct response: Stop the administration of the enema momentarily. Explanation: If the client reports abdominal discomfort or cramping, the nurse should momentarily stop the flow of solution. Increasing the flow of the enema may cause more cramping and discomfort. Cramping and discomfort are common complaints during enema administration so there is no need to notify the physician. Reference: Taylor, C., Lynn, P., & Bartlett, J., Fundamentals of Nursing, 9th ed., Philadelphia, Wolters Kluwer, 2019, Chapter 38: Bowel Elimination, p. 1455.

The nurse is slowly advancing a nasogastric (NG) tube when the client begins to gasp and is unable to vocalize. Which scenario has likely occurred?

Correct response: The NG tube is in the client's airway. Explanation: The tube is in the airway if the client shows signs of distress and cannot speak or hum. Excessive coughing and gagging may occur if the tube has curled in the back of throat. A vasovagal reaction is typically manifested by lightheadedness and fainting, not by gasping and an inability to vocalize. There is no indication that the client is forcefully resisting the procedure. Reference: Taylor, C., Lynn, P., & Bartlett, J., Fundamentals of Nursing, 9th ed., Philadelphia, Wolters Kluwer, 2019, Chapter 38: Bowel Elimination, pp. 1457-1462.

When reviewing a client's chart, which data related to a client experiencing diarrhea might suggest to the nurse a causative factor?

Correct response: The client returned from a foreign country 2 days ago. Explanation: Eating native food and drinking water in a foreign country may cause problems with digestion and elimination, such as diarrhea. To promote normal bowel elimination, people should drink 2,000 to 3,000 mL of fluids daily. Ignoring the urge to defecate and consuming large quantities of fiber, such as fresh vegetables, may lead to constipation. Reference: Taylor, C., Lynn, P., & Bartlett, J., Fundamentals of Nursing, 9th ed., Philadelphia, Wolters Kluwer, 2019, Chapter 38: Bowel Elimination, p. 1422.

A nurse is collecting a stool specimen from a client. Which measures are appropriate for this procedure? Select all that apply.

Correct response: The client should be asked to void first because the lab study may be inaccurate if the stool contains urine. The client should be instructed not to place toilet tissue in the bedpan or specimen container. Medical aseptic techniques are always followed. Handwashing is performed before and after glove use when handling a stool specimen. Explanation: The nurse would ask the client to void first and not to contaminate the stool with any urine. The nurse would ask the client not to place toilet paper in the stool specimen container or bedpan. The nurse would always follow medical aseptic techniques. The nurse would perform handwashing before and after glove use. The nurse would not ask the client to defecate directly into the toilet bowl. A specimen container would need to be placed in the commode. Usually 1 in (2.5 cm) of formed stool or 15 to 30 mL of liquid stool is sufficient for a stool specimen. Reference: Taylor, C., Lynn, P., & Bartlett, J., Fundamentals of Nursing, 9th ed., Philadelphia, Wolters Kluwer, 2019, Chapter 38: Bowel Elimination, p. 1428.

A nurse prepares to collect a stool sample from a client to test for fecal fat. Which guideline accurately describes a consideration in this process?

Correct response: The entire amount of stool produced for 24 to 72 hours should be sent to the laboratory. Explanation: For a timed stool test, such as fecal fat, the entire amount of stool produced for 24 to 72 hours is sent to the laboratory. In the cases of a room temperature sample or the presence of toilet paper, the nurse should call the laboratory to discuss possible effects on test results. Stool can be collected from an incontinent brief or diaper as long as it has not been contaminated by urine. Reference: Taylor, C., Lynn, P., & Bartlett, J., Fundamentals of Nursing, 9th ed., Philadelphia, Wolters Kluwer, 2019, Chapter 38: Bowel Elimination, p. 1429.

The newly hired graduate nurse is preparing to administer a cleansing enema. The nurse educator will intervene if which action is taken by the graduate nurse?

Correct response: The graduate places the client in Fowler's position. Explanation: Placing the client in Fowler's position during an enema will cause the solution to remain in the rectum; expulsion of the solution happens rapidly with minimal cleansing accomplished. The solution should be retained until the desired results are achieved. The solution should not be too hot or too cold, but administered at room temperature. Most people are uncomfortable about discussing the intestinal tract and bowel elimination, so this is an opportune time to discuss it. Reference: Taylor, C., Lynn, P., & Bartlett, J., Fundamentals of Nursing, 9th ed., Philadelphia, Wolters Kluwer, 2019, Chapter 38: Bowel Elimination, p. 1440.

The nurse is administering an oil-retention enema to a client. Which nursing actions in this procedure are performed correctly? Select all that apply.

Correct response: The nurse administers the oil-retention enema at body temperature. The nurse instructs the client to retain the oil for at least 30 minutes. The nurse administers a cleansing enema after the oil-retention enema. Explanation: Reference: Taylor, C., Lynn, P., & Bartlett, J., Fundamentals of Nursing, 9th ed., Philadelphia, Wolters Kluwer, 2019, Chapter 38: Bowel Elimination, p. 1440.

When caring for a client with a new colostomy, which assessment finding would be considered abnormal and would need to be reported to the physician?

Correct response: The stoma is prolapsed. Explanation: If the stoma is found to be prolapsed, the surgeon must be notified immediately. The stoma should be pink and remain on the abdominal surface. The mucosal tissue is fragile, so a small amount of bleeding may be normal. Reference: Taylor, C., Lynn, P., & Bartlett, J., Fundamentals of Nursing, 9th ed., Philadelphia, Wolters Kluwer, 2019, Chapter 38: Bowel Elimination, p. 1446.

The nursing student is performing a focused gastrointestinal assessment. Which action performed by the student would indicate to nurse faculty that further instruction is needed?

Correct response: The student sequenced from auscultation to inspection, and percussion to palpation. Explanation: The correct sequence for an abdominal assessment is inspection, then auscultation (done before palpation because palpation may disturb normal peristalsis and bowel motility), followed by percussion and palpation. The client should urinate before assessment and the knees should be flexed with the abdomen during the examination. Reference: Taylor, C., Lynn, P., & Bartlett, J., Fundamentals of Nursing, 9th ed., Philadelphia, Wolters Kluwer, 2019, Chapter 38: Bowel Elimination, p. 1426.

A nurse is assessing the stoma of a client with an ostomy. Which intervention should the nurse perform when providing peristomal care to the client to preserve skin integrity?

Correct response: Wash it with a mild cleanser and water. Explanation: Washing the stoma and surrounding skin with a mild cleanser and water and patting it dry can preserve skin integrity. When using a cleanser, it is important to rinse the area thoroughly. Any residue left on the skin can cause problems with the wafer adhering. Another way to protect the skin is to apply barrier substances such as karaya, a plant substance that becomes gelatinous when moistened, and commercial skin preparations around the stoma. Cleaning the stoma with just a dry, cotton bandage is not the correct way of preserving skin integrity. Reference: Taylor, C., Lynn, P., & Bartlett, J., Fundamentals of Nursing, 9th ed., Philadelphia, Wolters Kluwer, 2019, Chapter 38: Bowel Elimination, p. 1471.

Which factor is related to developmental changes in bowel habits for older adult clients?

Correct response: Weakened pelvic muscles lead to constipation. Explanation: Weakened pelvic muscles and decreased activity levels contribute to constipation in older adults. Increasing dietary fiber does not decrease peristalsis. Lactose intolerance is not a developmental change in older adults. Peeling fruit does not impact bowel habits in the older adults. Reference: Taylor, C., Lynn, P., & Bartlett, J., Fundamentals of Nursing, 9th ed., Philadelphia, Wolters Kluwer, 2019, Chapter 38: Bowel Elimination, p. 1421.

The nurse is presenting a lecture on ostomy bowel elimination at a community clinic. When questioned by the clients, which food would the nurse suggest as natural intestinal deodorizers?

Correct response: Yogurt and buttermilk Explanation: Buttermilk, parsley, and yogurt are foods that are natural intestinal deodorizers. Dried lentils, asparagus, turnips, fish, onions, and garlic are foods that produce odor. Reference: Taylor, C., Lynn, P., & Bartlett, J., Fundamentals of Nursing, 9th ed., Philadelphia, Wolters Kluwer, 2019, Chapter 38: Bowel Elimination, p. 1449.

For which client would digital removal of stool be contraindicated?

Correct response: a client recovering from prostate surgery Explanation: Digital removal of stool should not be performed on clients who have bowel inflammation or bowel infection, or after rectal, prostate, and colon surgery. None of the other listed health problems contraindicate digital removal of stool. Reference: Taylor, C., Lynn, P., & Bartlett, J., Fundamentals of Nursing, 9th ed., Philadelphia, Wolters Kluwer, 2019, Chapter 38: Bowel Elimination, p. 1441.

A nurse is caring for a client with constipation. The incidence of constipation tends to be high among clients who follow which diet?

Correct response: a diet lacking in fruits and vegetables Explanation: The incidence of constipation tends to be high among clients whose dietary habits lack sufficient raw fruits and vegetables, whole grains, seeds, and nuts, all of which contain adequate fiber. Dietary fiber, which becomes undigested cellulose, is important because it attracts water within the bowel, resulting in bulkier stool that is more quickly and easily eliminated. A diet lacking in glucose and water will cause dehydration first and then constipation, depending on other constituents of the diet. Diets consisting of whole grains, seeds, and nuts provide fiber, which helps in bowel movement. A diet lacking in meat and poultry products need not necessarily lead to constipation. Reference: Taylor, C., Lynn, P., & Bartlett, J., Fundamentals of Nursing, 9th ed., Philadelphia, Wolters Kluwer, 2019, Chapter 38: Bowel Elimination, p. 1423.

When assessing an elderly client for constipation, the nurse learns that the client uses mineral oil daily to relieve constipation. Which is an effect of prolonged use of mineral oil to relieve constipation?

Correct response: affects absorption of fat-soluble vitamins Explanation: Elderly clients who use mineral oil to prevent or relieve constipation need to be informed that prolonged use affects the absorption of fat-soluble vitamins such as A, D, E, and K. Bleeding and tissue trauma does not occur due to use of mineral oil for constipation but during the digital removal of faction. Use of mineral oil for constipation does not develop healthier bowel elimination patterns, but the use of bulk-forming products containing psyllium or polycarbophil does. Loss of elasticity in intestinal walls and slower motility is due to old age, not the use of mineral oil. Reference: Taylor, C., Lynn, P., & Bartlett, J., Fundamentals of Nursing, 9th ed., Philadelphia, Wolters Kluwer, 2019, Chapter 38: Bowel Elimination, p. 1435.

The risk for developing colorectal cancer during one's lifetime is 1 in 19. Nurses play an integral role in the promotion of colorectal cancer screening. What are risk factors for colorectal cancer? Select all that apply.

Correct response: age 50 and older a positive family history a history of inflammatory bowel disease Explanation: The risks for colorectal cancer increase after the age of 50, with a positive family history of colorectal cancer, and also with Crohn's disease. An important nursing responsibility is to teach clients about annual screening beginning at 50, encourage endoscopic exam every 5 years, or colonoscopy every 10 years for normal-risk individuals. Reference: Taylor, C., Lynn, P., & Bartlett, J., Fundamentals of Nursing, 9th ed., Philadelphia, Wolters Kluwer, 2019, Chapter 38: Bowel Elimination, p. 1428.

The proliferation of Clostridium difficile causes:

Correct response: antibiotic-associated diarrhea. Explanation: Normal intestinal flora inhibit the growth of Clostridium difficile. When broad-spectrum antibiotics, especially third-generation cephalosporins, are administered, normal flora is altered and C. difficile can proliferate and release toxins that cause antibiotic-associated diarrhea. Reference: Taylor, C., Lynn, P., & Bartlett, J., Fundamentals of Nursing, 9th ed., Philadelphia, Wolters Kluwer, 2019, Chapter 38: Bowel Elimination, pp. 1424-.

The nurse is evaluating stool characteristics of an adult client. Which color stool does the nurse identify as abnormal? Select all that apply.

Correct response: black clay colored yellow Explanation: The nurse identifies that normal stool varies in color from light to dark brown. Black, clay (tan) and yellow are considered abnormal colors for adult stool. Reference: Taylor, C., Lynn, P., & Bartlett, J., Fundamentals of Nursing, 9th ed., Philadelphia, Wolters Kluwer, 2019, Chapter 38: Bowel Elimination, p. 1427.

A client wishes to increase fiber to promote more regular bowel movements. Which food will the nurse recommend that the client consume?

Correct response: brown rice Explanation: The nurse will recommend brown rice, a food that is high in dietary fiber. Other selections listed do not contain high fiber. Reference: Taylor, C., Lynn, P., & Bartlett, J., Fundamentals of Nursing, 9th ed., Philadelphia, Wolters Kluwer, 2019, Chapter 38: Bowel Elimination, pp. 1422-1423.

The nurse is educating a new colostomy client on gas-producing foods. Which food is a gas-producing food the client may choose to avoid?

Correct response: brussels sprouts Explanation: Certain foods (e.g., cabbage, onions, legumes) often increase the amount of flatus produced in the intestine. Reference: Taylor, C., Lynn, P., & Bartlett, J., Fundamentals of Nursing, 9th ed., Philadelphia, Wolters Kluwer, 2019, Chapter 38: Bowel Elimination, p. 1449.

A student nurse studying human anatomy knows that a structure of the large intestine is the:

Correct response: cecum Explanation: The small intestine consists of the duodenum, jejunum, and ileum. The large intestine consists of the cecum, colon (ascending, transverse, descending, and sigmoid), and rectum. Reference: Taylor, C., Lynn, P., & Bartlett, J., Fundamentals of Nursing, 9th ed., Philadelphia, Wolters Kluwer, 2019, Chapter 38: Bowel Elimination, p. 1419.

The nurse is evaluating stool characteristics of an adult client. Which of the following would describe a normal stool? Select all that apply.

Correct response: dark brown light brown Explanation: The nurse identifies that normal stool varies in color from light to dark brown. Black, clay (tan), and yellow are considered abnormal colors for adult stool. Reference: Taylor, C., Lynn, P., & Bartlett, J., Fundamentals of Nursing, 9th ed., Philadelphia, Wolters Kluwer, 2019, Chapter 38: Bowel Elimination, p. 1427.

The nurse is scheduling tests for a client who is experiencing bowel alterations. What is the most logical sequence of tests to ensure an accurate diagnosis?

Correct response: fecal occult blood test, barium studies, endoscopic examination Explanation: There is a specific sequence that bowel tests must be performed due to the results of certain contrasts and other preps that must be given. The nurse would verify that the tests are done in the correct order: Fecal occult blood test, barium studies, and then endoscopic examination. Reference: Taylor, C., Lynn, P., & Bartlett, J., Fundamentals of Nursing, 9th ed., Philadelphia, Wolters Kluwer, 2019, Chapter 38: Bowel Elimination, p. 1432.

The nurse will gather which type of solution to administer a cleansing enema to a client who needs to have water drawn into the bowel?

Correct response: hypertonic saline Explanation: The nurse will gather a hypertonic solution to draw water into the bowel by irritating local tissues. Mineral oil is used for lubrication and softening of stool. Tap water is used to distend the rectum and moisten stool; soap and water are used to do the same plus irritate local tissue. Reference: Taylor, C., Lynn, P., & Bartlett, J., Fundamentals of Nursing, 9th ed., Philadelphia, Wolters Kluwer, 2019, Chapter 38: Bowel Elimination, p. 1439.

A cleansing enema has been ordered for the client to draw water into the bowel. Which type of solution does the nurse gather?

Correct response: hypertonic saline Explanation: The nurse will gather a hypertonic solution, which is used to irritate local tissue and draw water into the bowel. Mineral oil is used for lubrication and softening of stool. Tap water is used to distend the rectum and moisten stool; soap and water are used to do the same plus irritate local tissue. Reference: Taylor, C., Lynn, P., & Bartlett, J., Fundamentals of Nursing, 9th ed., Philadelphia, Wolters Kluwer, 2019, Chapter 38: Bowel Elimination, pp. 1439-1440.

Which foods will the nurse recommend to avoid for a client with uncomfortable, frequent episodes of flatulence? Select all that apply.

Correct response: lentils onions cabbage Explanation: Lentils, onions, and cabbage are known to produce gas. Meats are generally not associated with formation of gas. Reference: Taylor, C., Lynn, P., & Bartlett, J., Fundamentals of Nursing, 9th ed., Philadelphia, Wolters Kluwer, 2019, Chapter 38: Bowel Elimination, p. 1437.

The type of stool that will be expelled into the ostomy bag by a client who has undergone surgery for an ileostomy will be:

Correct response: liquid consistency. Explanation: Stool produced from an ileostomy is liquid and contains large quantities of electrolytes. Reference: Taylor, C., Lynn, P., & Bartlett, J., Fundamentals of Nursing, 9th ed., Philadelphia, Wolters Kluwer, 2019, Chapter 38: Bowel Elimination, p. 1445.

Digital rectal examination confirms that a client has an impaction, and an enema solution has been ordered to lubricate the stool and intestinal mucosa without distending the intestine. What solution best meets this client's needs?

Correct response: oil Explanation: Mineral, olive, or cottonseed oil is used to lubricate the stool and intestinal mucosa without distending the intestine. Water and normal saline do not have these qualities. Soap has lubricant properties but primarily acts by irritating the intestinal mucosa. Reference: Taylor, C., Lynn, P., & Bartlett, J., Fundamentals of Nursing, 9th ed., Philadelphia, Wolters Kluwer, 2019, Chapter 38: Bowel Elimination, p. 1439.

The nurse is administering a rectal suppository. How far will the nurse insert the suppository?

Correct response: past the internal sphincter Explanation: To be effective, a suppository must be inserted past the internal sphincter, which is about the distance of the finger of insertion. Reference: Taylor, C., Lynn, P., & Bartlett, J., Fundamentals of Nursing, 9th ed., Philadelphia, Wolters Kluwer, 2019, Chapter 38: Bowel Elimination, p. 1443.

When caring for a client with fecal incontinence, the nurse knows that fecal incontinence is the result of:

Correct response: physiologic or lifestyle changes in the client. Explanation: Fecal incontinence mainly results from physiologic or lifestyle changes that impair muscle activity and sensation of the gastrointestinal tract. Particularly in the older adult, the weakening of the intestinal walls and decreased muscle tone can lead to bowel incontinence. Reference: Taylor, C., Lynn, P., & Bartlett, J., Fundamentals of Nursing, 9th ed., Philadelphia, Wolters Kluwer, 2019, Chapter 38: Bowel Elimination, p. 1421.

The health care provider prescribes a large-volume cleansing enema for a client. What outcome does the nurse identify that will be optimal for this client?

Correct response: removes hardened fecal impactions from the rectum Explanation: Cleansing enemas are given to remove feces from the colon. Some of the reasons for administering a cleansing enema include relieving constipation or fecal impaction; preventing involuntary escape of fecal material during surgical procedures; promoting visualization of the intestinal tract by radiographic or instrument examination; and helping to establish regular bowel function during a bowel training program. Oil-retention enemas lubricate the stool and intestinal mucosa, making defecation easier. Enemas are not used for diarrhea. Reference: Taylor, C., Lynn, P., & Bartlett, J., Fundamentals of Nursing, 9th ed., Philadelphia, Wolters Kluwer, 2019, Chapter 38: Bowel Elimination, p. 1481.

The nurse is caring for a client with constipation related to a small bowel obstruction. How will the nurse document this finding?

Correct response: secondary constipation Explanation: The nurse will document this finding as secondary constipation, which is a consequence of a pathologic disorder. The other answers do not correctly describe the client's condition. Reference: Taylor, C., Lynn, P., & Bartlett, J., Fundamentals of Nursing, 9th ed., Philadelphia, Wolters Kluwer, 2019, Chapter 38: Bowel Elimination, p. 1434.

The nurse is caring for an older adult client with diarrhea. Which assessment finding requires immediate nursing intervention?

Correct response: skin turgor response 5 seconds Explanation: Skin turgor response that is greater than 3 seconds, especially in an older adult clients, requires nursing intervention. Older adults with diarrhea can more easily become dehydrated and develop fluid and electrolyte imbalances. Other assessment findings are normal. Reference: Taylor, C., Lynn, P., & Bartlett, J., Fundamentals of Nursing, 9th ed., Philadelphia, Wolters Kluwer, 2019, Chapter 38: Bowel Elimination, p. 1436

A client has received nursing teaching about proper skin care at a stomal site. The nurse's teaching has been effective when the client identifies which solution is used to clean the stoma?

Correct response: water and mild soap Explanation: The nurse will teach the client to use water and mild soap to cleanse the stoma. Saline only will not provide cleansing; an alcohol-based sanitizer will dry the stoma; mineral oil is not appropriate for cleansing. Reference: Taylor, C., Lynn, P., & Bartlett, J., Fundamentals of Nursing, 9th ed., Philadelphia, Wolters Kluwer, 2019, Chapter 38: Bowel Elimination, p. 1471.

A nurse is caring for a client with fecal impaction. Which factors cause fecal impaction? Select all that apply.

Correct response: weak abdominal muscles severe dehydration unrelieved constipation Explanation: Fecal impaction results from unrelieved constipation, retained barium from an intestinal x-ray, dehydration, and weakness of abdominal muscles. Clients with a fecal impaction usually report a frequent desire to defecate but an inability to do so. Insufficient mastication of food does not lead to fecal impaction but may sometimes lead to indigestion and temporary constipation. Excess intake of fibrous food, such as raw fruits and vegetables, does not lead to constipation; instead, adequate fiber helps facilitate bowel movement and elimination. Reference: Taylor, C., Lynn, P., & Bartlett, J., Fundamentals of Nursing, 9th ed., Philadelphia, Wolters Kluwer, 2019, Chapter 38: Bowel Elimination, p. 1434.

A client with no significant medical history reports constipation for the past week. Which assessment information will the nurse collect? Select all that apply.

Correct response: whether the client is taking new medication the client's normal bowel habits if the client feels a sensation of rectal fullness if the client has used laxatives in the past Explanation: Reference: Taylor, C., Lynn, P., & Bartlett, J., Fundamentals of Nursing, 9th ed., Philadelphia, Wolters Kluwer, 2019, Chapter 38: Bowel Elimination, p. 1425.

A bowel training program includes which of the following? A) Using a diet that is low in bulk B) Decreasing fluid intake to 1000 mL C) Administering an enema once a day to stimulate peristalsis D) Allowing ample time for Evacuation

D) Allowing ample time for Evacuation For a bowel training program to be effective, the patient must have ample time for evacuation usually 20 to 30 minutes. Fluid intake is increased to 2500 to 3000 mL, food high in bulk is recommended as part of the program: and a daily enema is not administered in bowel training program. A cathartic Suppository maybe use 30 minutes before the patients usually defecation time to stimulate peristalsis

Which of the following is an appropriate nursing action to promote regular bowel habits? A) Encourage the patient to avoid moving his bowels until a certain time of day B) Encourage the patient to avoid excess fluid intake and too much fiber C) Avoid strenuous exercise to limit stress on the abdominal muscles and impair peristalsis D) Assisting the patient to a normal position as possible to defecate

D) Assisting the patient to a normal position as possible to defecate Sitting upright on a toilet or commode promotes defecation. If the patient must use a bedpan, raise the head of the bed 30 to 45°. Patient should be encouraged to move their bowels at their usual time of the day. However, the patient should not be encouraged to put off defecation if the urge arises before or after their usual time. Patient should be encouraged to consume 2000 to 3000 mL of fluid, preferably water, and increase fiber, to promote regular defecation. Regular exercise improves gastrointestinal activities and aids in defecation

As the nurse prepares to assist Mrs. P with her newly created Ileostomy, She is aware of which of the following? A) An appliance will not be required on the continual basis B) The size of the stoma stabilizes within two weeks C) Irrigation is necessary for regulation D) Fecal drainage will be liquid

D) Fecal drainage will be liquid And appliance is usually required on a continual basis because the fecal drainage is liquid. Stomas size usually stabilizes within 4 to 6 weeks, and Ileostomy Irrigation is not necessary because fecal matter is liquid

Which one of the following enemas would be used for a patient with intestinal parasites? A. Oil-retention enema B. Carminative enema C. Nutritive enema D. Anthelmintic enema

D. Anthelmintic enema Rationale: Antihelmintic enemas destroy intestinal parasites.

*While a nurse is administering a cleansing enema, the client reports abdominal cramping. Which of the following actions should the nurse take? A. Have the client hold his breath briefly and bear down B. Discontinue the fluid instillation C. Remind the client that cramping is common at this time D. Lower the enema fluid container

D. Lower the enema fluid container To relieve the client's discomfort, the nurse should slow the rate of instillation by reducing the height of the enema solution container.

A nurse is caring for a client who will perform fecal occult blood testing at home. Which of the following information should the nurse include when explaining the procedure to the client? A. Eating more protein is optimal prior to testing. B. One stool specimen is sufficient for testing. C. A red color change indicates a positive test. D. The specimen cannot be contaminated with urine.

D. The specimen cannot be contaminated with urine. For fecal occult blood testing, the nurse should warn the client not to contaminate the stool specimens with water or urine.

What primary risk does the nurse need to assess for among patients who use laxatives long term?

Dependance

Which of the following symptoms are known side effects of antibiotics? a) Fecal impaction b) Abdominal bloating c) Diarrhea d) Constipation

Diarrhea

Which of the following is a true statement about the effects of medication on bowel elimination?

Diarrhea commonly occurs with ammoxicillin clavulanate use.

A nurse is caring for a client with constipation. The incidence of constipation tends to be high among clients taking which of the following diets? a) A diet consisting of whole grains, seeds, and nuts b) A diet lacking in fruits and vegetables c) A diet lacking in meat and poultry products d) A diet lacking in glucose and water

Diet lacking in fruits and vegs

Which of the following statements best explains why digital removal of stool is considered a last resort after other methods of bowel evacuation have been unsuccessful? a) It often causes rebound diarrhea and electrolyte loss. b) Digital removal of stool may cause parasympathetic stimulation. c) Nurses find the procedure distasteful and difficult to perform. d) Most patients will not consent to have digital removal of stool.

Digital removal of stool may cause parasympathetic stimulation

Which of the following statements best explains why digital removal of stool is considered a last resort after other methods of bowel evacuation have been unsuccessful?

Digital removal of stool may cause parasympathetic stimulation. Explanation: The procedure may stimulate a vagal response, which increases parasympathetic stimulation.

The nurse is preparing to auscultate the bowel sounds of a patient with a nasogastric tube in place to low intermittent suction. How shall the nurse approach the assessment of bowel sounds and manage the nasogastric tube?

Disconnect the nasogastric tube from suction during the assessment of bowel sounds.

The nurse is preparing to auscultate the bowel sounds of a patient with a nasogastric tube in place to low intermittent suction. How shall the nurse approach the assessment of bowel sounds and manage the nasogastric tube? a) Disconnect the nasogastric tube from the suction for 1 hour prior to the assessment of bowel sounds. b) Apply continuous suction to the nasogastric tube during assessment of bowel sounds. c) Disconnect the nasogastric tube from suction during the assessment of bowel sounds. d) Allow the low intermittent suction to continue during the assessment of bowel sounds.

Disconnect the nasogastric tube from suction during the assessment of bowel sounds.

Which of the following would be a common nursing diagnosis for the patient with an ileostomy?

Disturbed body image

Percussion of Abdomen

EXPECT resonant or tympany - Resonance/Tympany: normal - Dullness: fluid, mass, tumor -Hyperresonance: excess fluid -Gurgling: normal and can be heard when auscultating

A nurse is caring for a client with diarrhea. Which of the following interventions can help provide relief to a client with diarrhea?

Encourage a clear liquid diet

A patient who is postoperative day 1 has rung the call light twice during the nurse's shift in order to request assistance transferring to a bedside commode. In both cases, however, the patient has been unable to defecate. In light of the fact that the patient's last bowel movement was the morning of surgery, what action should the nurse first take? a) Obtain a diet change order to increase the amount of fiber in the patient's meals b) Facilitate a more private setting, such as assisting the patient to a bathroom c) Administer a normal saline enema after obtaining the relevant order d) Position the patient on their side and administer a glycerin suppository

Facilitate a more private setting, such as assisting the patient to a bathroom

As the nurse prepares to assist Mrs. P with her newly created ileostomy, she is aware of which of the following?

Fecal drainage will be liquid.

Which one of the following is a recommended food for an older adult who is constipated? A. Cheese B. Fruit C. Cabbage D. Eggs

Fruit

What is a good food to help with constipation?

Fruits and vegetables! Cheese and eggs = constipating effect Cabbage = produces gas

A nurse is assessing a client with constipation and severe rectal pain. Which of the following actions should the nurse perform to determine the presence of fecal impaction?

Inserted a lubricated, gloved finger into the rectum.

Physical Inspection of Anus and Rectum

Inspect and Palpate - cracks, nodules, distended veins, fecal mass (may need to disimpact) - insert gloved finger to assess sphincter tone and smoothness of lining - inspect perineal area for irritation r/t diarrhea

Mr. J has a fecal impaction. The nurse correctly administers an oil-retention enema by doing which of the following?

Instructing the patient to retain the enema for at least 30 minutes.

Which of the following medications causes constipation? a) Iron supplements b) Aspirin c) Dulcolax d) Magnesium antacids

Iron supplements

During the physical examination of a client, the nurse oercusses the abdomen. In which abdominal quadrant should the nurse expect to hear tympany? a) Right lower quadrant b) Right upper quadrant c) Left upper quadrant d) Left lower quadrant

LUQ

A patient's last bowel movement was 4 days ago and oral laxatives and dietary changes have failed to prompt a bowel movement. How should the nurse position the patient in anticipation of administering a cleansing enema?

Left side-lying

A patient's last bowel movement was 4 days ago and oral laxatives and dietary changes have failed to prompt a bowel movement. How should the nurse position the patient in anticipation of administering a cleansing enema? a) Right side-lying b) Prone c) Left side-lying d) Supine

Left side-lying

The type of stool that will be expelled into the ostomy bag by a client who has undergone surgery for an ileostomy will be

Liquid

The type of stool that will be expelled into the ostomy bag by a client who has undergone surgery for an ileostomy will be a) Soft semi-formed b) Bloody c) Liquid consistency d) Mucus filled

Liquid consistency

A nurse is providing care to a client who has undergone a colonoscopy. Which of the following would be most appropriate for the nurse to do after the procedure? a) Limit oral fluid intake b) Monitor for rectal bleeding c) Avoid giving solid food d) Administer a laxative to the client

Monitor for rectal bleeding

A nurse is providing care to a client who has undergone a colonoscopy. Which of the following would be most appropriate for the nurse to do after the procedure?

Monitor for rectal bleeding Explanation: The nurse should monitor the client for rectal bleeding after a colonoscopy

Which of the following enema solutions lubricates the stool and intestinal mucosa without distending the intestine? a) Oil b) Soap c) Water d) Normal saline

Oil

Colostomy

Opening into abdominal wall for fecal elimination

When explaining the action of a hypertonic solution enema, the nurse incorporates which of the following as a basis for action?

Osmosis of water into colon.

During a home visit, the nurse learns that the client ensures a daily bowel movement with the help of laxatives. The client feels that deviation from a bowel movement every day is unhealthy. Which nursing diagnosis would the nurse most likely identify?

Perceived contipation

A nurse is caring for a client with an abdominal injury at a health care facility. The client tells the nurse that he passed blood-stained stool. Which of the following nursing actions is appropriate when a client reports blood in the stool? a) Save a sample of the stool in a container. b) Report about the stool to the physician. c) Send the stool sample to the laboratory. d) Perform a screening test on stool samples.

Perform a screening test on stool samples

A barium enema should be done before an upper gastrointestinal series because of which of the following? +

Retained barium may cloud the colon.

Types of Colostomies

Sigmoid, Descending, Transverse, Ascending: Lg Intestine Ileostomy: Small intestine = more watery than others **all can be reversed * temp or permanent * the disease portion is removed and colostomy performed

https://www.studyblue.com/notes/note/n/prepu-2/deck/5391415

Study Blue

Nurses should recommend avoiding the habitual use of laxatives. Which of the following is the rationale for this?

They will cause chronic constipation.

When collecting stool using the technique "timed specimen," the nurse should consider the first stool passed by the patient as the start of the collection period. A. True B. False

True

Indirect Visualization Studies

Upper GI AND Small bowel series Barium enema Abdominal ultrasound Magnetic resonance imaging Abdominal CT scan

Mr. T is nervous about a colonoscopy scheduled for tomorrow. The nurse describes the test by explaining that it allows which of the following?

Visual examination of the large intestine.

A nurse is assessing the stoma of a client with an ostomy. Which of the following interventions should the nurse perform when providing peristomal care to the client to preserve skin integrity?

Wash with soap and water

Which of the following factors is related to developmental changes in bowel habits for elderly clients?

Weakened pelvic muscles lead to constipation

hemorrhoids

abnormally distended rectal veins.

Nutritive enema

administer fluids and nutrition rectally

colostomy

an opening into the colon that permits feces to exit through the stoma.

stoma

artificial opening for waste excretion located on the body surface.

Diagnostic: Barium Enema

barium enema and xrays PREP: NPO, clear diet, enema, consent, review hx for contraindications for enema (gi bleed, colitis) AFTER: fluids, barium may lighten stool, encourage rest

valsalva maneuver consideration

bearing down -increase intrabdominal pressure results in decreased blood flow to the heart and decreases cardiac output -dangerously elevates BP in an already hypertensive individual and contraindicated in patients with CV problems

iron salts

black stool

obstruction

blockage of colon -intestinal -mechanical -functional

occult blood

blood present in such minute quantities that it cannot be detected with the unassisted eye.

NORMAL STOMA:

bright pink, red no black, brown, white changes indicated profusion problem alert MD immediately should be be prolapsed or retracted

metamucil

bulk forming -draws water into stool and stimulates peristalsis

infants

characteristics of stool and frequency depend on formula or breast milk

what food should a patient avoid if they are constipated

cheese

constipating foods

cheese meat eggs pasta

fecal impaction

collection in the rectum of hardened feces that cannot be passed -common in older adults -distended abdomen, oozing feces, pain -chronic constipation -may need digital removal -enema -suppository

fecal impaction

collection in the rectum of hardened feces that cannot be passed.

older adult

constipation is often a chronic problem; diarrhea and fecal incontinence may result from physiologic or lifestyle changes

Peristalsis

controlled by nervous system every 3-12 minutes mass sweeps 1-4 times per 24 hour period 1/3 to 1/2 of food waste excreted in stool within 24 hours

A patient's recent diagnosis of colorectal cancer has required a hemicolectomy (removal of part of the bowel) and the creation of a colostomy. The nurse would recognize that the patient's stoma is healthy when it appears: a) dark pink and moist. b) red and dry. c) dark or purple-blue. d) off-white or pale pink.

dark pink and moist.

Antidiarrheals: action on GI smooth muscle

decreases peristalsis and motility may be addictive causes drowsiness ex: opium, loperamide, diphenoxylate

child,adolescent, adult

defecation patterns vary in quantity and frequency and rhythmicity

Antihelmintic enema

destroy intestinal parasites

endoscopy

direct visualization of hollow organs of the body using an endoscope or flexible, lighted tube.

hemorrhoids

distended veins from the vertical folds in the rectum

laxative

drug used to induce emptying of the intestinal tract.

mineral oil

emollient -lubricated intestines and blocks colonic water absorption to soften stool and make easier to pass

defecation

emptying of the intestinal tract; synonym for bowel movement.

Antidiarrheals: antisecretory/microbial

ex: bismuth subsalicylate decreases GI tract secretion no drowsiness, may decrease absorption of other meds

Emollient/Stool Softener

ex: colace allows water and fat to penetrate and lubricate the stool for people that can't strain may interfere with fat soluble vitamins

stimulant

ex: ducolax promotes peristalsis works quickly most abused on market, lazy bowel syndrome, affects vitamin d absorption, not recommeded for eldery

saline-osmotic

ex: fleet enema drug draws water into intestine and stimulates peristalsis used when rapid cleansing desired can produce dehydration, not for elderly, kidney disease

Antidiarrheals: Adsorbent

ex: kaolin-pectin absorbs and soothes no drowsiness interferes with adsorbion for other medicines

Bulk Forming Laxative

ex: metamucil stool absorbs water and swells and stimulates peristalsis works quickly - 24 hours

Lubricant

ex: mineral oil absorbed from intestinal tract and softens stool for passing usually effective within 8 hours may interefere with fat soluble vitamins absorption

Diagnostic: Sigmoidoscopy

exam of distal sigmoid, rectum, anal through scope PREP: consent, enemas and light food, no sedation AFTER: gas normal, observe for perforation, bleeding normal with biopsy

Diagnostic: Colonoscopy

exam of rectum, colon, small bowl by lighted scope PREP: consent, bowel prep/clear liquid diet, sedation AFTER: gas is normal, usual diet, vitals, observe for rectal bleeding, pain, fever

Sigmoidoscopy

examines the distal sigmoid colon, rectum, and anal canal through a flexible or rigid sigmoidoscope

Esophagogastoduodenoscopy

examines the esophagus, stomach, and upper duodenum through an optic scope

flatulence

excessive formation of gases in the GI tract -harris flush

flatulence

excessive formation of gases in the gastrointestinal tract.

stool

excreted feces.

An elderly client who is wheelchair bound following a cerebrovascular accident is being assessed by the nurse. The nurse notes the client has seepage of stool from the anus. The nurse knows this is indicative of a) Fecal impaction b) Diarrhea c) Intestinal infection d) Constipation

fecal impaction

UGI series

fluoroscopy -barium sulfate contrast -NPO after midnight the day of -post test cathartic to prevent fecal impaction

Valsalva maneuver

forcible exhalation against a closed glottis, resulting in increased intrathoracic pressure.

laxative foods

fruits and vegetables bran chocolate coffee alcohol

ostomy

general term referring to an artificial opening, usually used to refer to an opening created for the excretion of body wastes.

antibiotics

green-gray color

Carminative enema

help expel flatus from the rectum

Colstomy Care

help the pt to keep it as free from odor as possible help with body image pay attention to your facial expression when cleaning it not the size, skin around (intact and dry and clean) measure I&O encourage pt to look at ostomy

tympany, the normal sound percussed over the abdomen, is caused by what

hollow organs

bowel incontinence

inability of the anal sphincter to control the discharge of fecal and gaseous material -skin barrier -change and clean frequently

Diagnostic: Upper GI and Small Bowel Series

ingestion of barium sulfate and fluoroscopic exam PREP: consent, NPO, barium tastes chalky todrink AFTER: post test cathartic to prevent fecal impaction, stools may be lighter for several days

flatus

intestinal gas.

feces

intestinal waste products.

enema

introduction of solution into the lower bowel.

fecal incontinence

involuntary or inappropriate passing of stool or flatus.

peristalsis

involuntary, progressive wave-like movement of the musculature of the gastrointestinal tract.

UGI series

involved fluoroscopic exam of the esophagus, stomach, and small intestine after ingestion of barium sulfate

CT

iodine allergy -kidney function -NPO 4 hr prior -NO METFORMIN (to prevent renal failure)

ileostomy

liquid fecal content from the ileum or the small intestine to be eliminated through the stoma

Oil retention enema

lubricate the stool and intestinal mucosa making defecation easier

Bowel Training Progam

manipulate's factors for the patient to form a soft stool at regular intervals based on diet, exercise, fluids, and time for defecation

MRI

may need fasting -metal removed -contrast monitoring after

cathartic

medication that strongly increases gastrointestinal motility and promotes defecation.

Abdominal ultrasound

no radiation -NPO 8 hr prior -before barium tests

isotonic enema

normal saline -500-1000 mL -distend intestine, increase peristalsis, soften stool -15 min -sodium retention F&E imbalance

gas-producing foods

onions cabbage beans cauliflower

anus

opening at the end of the anal canal.

ileostomy

opening into the small intestine allows fecal content from the ileum to be eliminated through the stoma.

suppository

oval- or cone-shaped substance that is inserted into a body cavity and that melts at body temperature.

paralytic ileus

paralysis of intestinal peristalsis.

stoma

part of the ostomy attached to the skin

diarrhea

passage of a liquid and informed stool ->3 loose stools daily -F&E imbalances and dehydration -antidiarrheal and rehydration -many possible causes

constipation

passage of dry hard fecal material -high risk: bedridden with opioids, low fluid and fiber, depression, local lesion or CNS disease -20-30g fiber and 2,000-3,000mL daily of fluid -laxatives

constipation

passage of dry, hard, fecal material.

diarrhea

passage of liquid and unformed stools.

During a home visit, the nurse learns that the client ensures a daily bowel movement with the help of laxatives. The client feels that deviation from a bowel movement every day is unhealthy. Which nursing diagnosis would the nurse most likely identify? a) Constipation b) Perceived constipation c) Bowel incontinence d) Risk of constipation

perceived constipation

colostomy

permits formed feces in colon to exit through the stoma

toddler

physiologic maturity is first priority for bowel training

bowel training program

program that manipulates factors within a person's control to produce a regular pattern of comfortable defecation without medication or enemas.

NG tube

purpose: decompress/ drain stomach feed with peristalsis absent GI tract to rest/heal before/after surgery

Prevent food poisoning

refrigerator leftover within 2 hours of eating only pasteurized juice to children

milk of magnesia

saline-osmotic -draws water into intestines and increases peristalsis

Diagnostic: Abd Ultrasound

sonogram PREP: painless, NPO AFTER: normal diet and fluids

dulcolax

stimulant -promote peristalsis by irritating or stimulation the intestinal mucosa and nerve endings

aspirin or anticoagulants

stool is pink, red, or black

colace

stool softener -detergent activity that allows water and fat to penetrate and lubricate the stool

ostomy

surgically formed opening from inside of an organ to the outside

hypotonic enema

tap water -500-1000 mL -distend intestine, increase peristalsis, soften stool -15 min -F&E imbalance and water intoxication

Time stool specimen

the first stool passed by the patient is the start of the collection period

bowel incontinence

the inability of the anal sphincter to control the discharge of fecal and gaseous material.

Diagnostic: Esophagogastroduodenoscopy

visual exam of esophagus, stomach, upper duodenum by lighted scope PREP: NPO, consent, dentures out, local anesthetic into mouth AFTER: sore throat normal; check vitals, no food til gag reflex returns, observe for perforation, saline gargles may help

Colonoscopy

visualizeds the rectum, colon, bowel using a lighted scope

Which of the following factors is related to developmental changes in bowel habits for elderly clients? a) Increase in dietary fiber can decrease peristalsis b) Weakened pelvic muscles lead to constipation c) The elderly should peel fruits before eating d) Milk products cause constipation in lactose intolerance

weakened pelvic muscles lead to constipation

antacids

white discoloration or speckling in stool

Diagnostic: Abd CT Scan

xrays PREP: oral contrast: check for allergies, BUN/creatinine, NPO, informed consent, metformin must be stopped prior to study and held after to prevent renal failure AFTER: monitor for IV/contrast reactions


Set pelajaran terkait

Saunders-ATI-Davis-Intrapartum Study Guide

View Set

Abeka Investigating God's World Chapter 5 Test

View Set

Exam 1 multiple choice questions

View Set

Sonidos en Contexto 20 [f] 21 [t͡ʃ] and 22 [m] [ɱ] [n̪] [n] [ɲ] [ŋ]

View Set

Series 6 Section 1 Debt Securities

View Set